Prove Convergence of $\sum_{n=1}^{\infty} a_n$ and $\sum_{n=1}^{\infty} b_n$

In summary: Yes, you are right. I was starting at n=0. Sorry. And I haven't even thought about going the other way yet. But I'm not going to do that until tomorrow. Yawn! That gives you some time to think about it and beat me to the... (solution).In summary, the conversation discusses a proof that shows the convergence of two series ##\sum_{n=1}^{\infty} a_n## and ##\sum_{n=1}^{\infty} b_n## are equivalent. The forward direction is shown to be relatively easy, but the reverse direction is still being worked on. The conversation also includes a discussion on how to reindex the series and a
  • #1
mahler1
222
0
Homework Statement .

Let ##\{a_n\}_{n \in \mathbb N}## a sequence of real numbers such that ##lim_{n \to \infty} a_n=0## and let ##b_n=a_n+2a_{n+1}-a_{n+2}##.

Prove that ##\sum_{n=1}^{\infty} a_n## is convergent iff ##\sum_{n=1}^{\infty} b_n## is convergent.

The attempt at a solution.

Honestly, I don't have a clue how to prove this. I know that if ##\{a_n\}_{n \in \mathbb N}## is convergent, then for a given ##ε>0##, there exists ##N : m,n>N## (suppose ##n>m##) ##\implies |\sum_{i=1}^ n a_i -\sum_{i=1}^ m a_i|=|a_{m+1}+...+a_n|<\epsilon##. I've tried to relate this to partial sums of the series ##\sum_{n=1}^{\infty} b_n## but I couldn't conclude anything. With the other implication I am also stuck. And I don't see how to use the fact that ##lim_{n \to \infty} a_n=0##.
 
Physics news on Phys.org
  • #2
mahler1 said:
Homework Statement .

Let ##\{a_n\}_{n \in \mathbb N}## a sequence of real numbers such that ##lim_{n \to \infty} a_n=0## and let ##b_n=a_n+2a_{n+1}-a_{n+2}##.

Prove that ##\sum_{n=1}^{\infty} a_n## is convergent iff ##\sum_{n=1}^{\infty} b_n## is convergent.

The attempt at a solution.

Honestly, I don't have a clue how to prove this. I know that if ##\{a_n\}_{n \in \mathbb N}## is convergent, then for a given ##ε>0##, there exists ##N : m,n>N## (suppose ##n>m##) ##\implies |\sum_{i=1}^ n a_i -\sum_{i=1}^ m a_i|=|a_{m+1}+...+a_n|<\epsilon##. I've tried to relate this to partial sums of the series ##\sum_{n=1}^{\infty} b_n## but I couldn't conclude anything. With the other implication I am also stuck. And I don't see how to use the fact that ##lim_{n \to \infty} a_n=0##.

You really shouldn't be stuck on the forward direction. If ##\sum_{n=1}^{\infty} a_n=L## then what are ##\sum_{n=1}^{\infty} a_{n+1}## and ##\sum_{n=1}^{\infty} a_{n+2}##? So it should be pretty easy to guess what ##\sum_{n=1}^{\infty} b_n## is and prove it using partial sums. The reverse direction is the harder one. That's where you need ##lim_{n \to \infty} a_n=0##. If not then a counterexample is ##a_n=1## for all n. Then ##\sum_{n=1}^{\infty} b_n## converges but ##\sum_{n=1}^{\infty} a_n## doesn't.
 
  • Like
Likes 1 person
  • #3
Dick said:
You really shouldn't be stuck on the forward direction. If ##\sum_{n=1}^{\infty} a_n=L## then what are ##\sum_{n=1}^{\infty} a_{n+1}## and ##\sum_{n=1}^{\infty} a_{n+2}##? So it should be pretty easy to guess what ##\sum_{n=1}^{\infty} b_n## is and prove it using partial sums. The reverse direction is the harder one. That's where you need ##lim_{n \to \infty} a_n=0##. If not then a counterexample is ##a_n=1## for all n. Then ##\sum_{n=1}^{\infty} b_n## converges but ##\sum_{n=1}^{\infty} a_n## doesn't.

It's true, the forward direction wasn't difficult:

Lets prove that ##\sum_{n=1}^{\infty} b_n=2L-a_1+a_2##

By hypothesis, ##\sum_{n=1}^{\infty} a_n## is convergent. Call its limit ##L##, then given ##\epsilon>0##, there exists ##N_{\epsilon} : \forall n\geq N_{\epsilon} \implies |\sum_{i=1}^n a_i -L|<\dfrac{\epsilon}{3}##.

Then, for ##n\geq N_{\epsilon}##, ##|\sum_{i=1}^n b_i -(2L-a_1+a_2)|=|\sum_{i=1}^n a_i + 2\sum_{i=1}^n a_{i+1}-\sum_{i=1}^n a_{i+2}-(2L-a_1+a_2)|##. By an index change of the form ##j=i+1## and ##k=i+2##, we have

##|\sum_{i=1}^n a_i + 2\sum_{i=1}^n a_{i+1}-\sum_{i=1}^n a_{i+2}-(2L-a_1+a_2)|=|\sum_{i=1}^n a_i + 2\sum_{j=1}^{n+1} a_j-2a_1-\sum_{k=1}^{n+2} a_k+ a_1+a_2-(2L-a_1+a_2)|=|\sum_{i=1}^n a_i-L + 2\sum_{j=1}^{n+1} a_j-2L+L-\sum_{k=1}^{n+2} a_k|\leq |\sum_{i=1}^n a_i-L|+2|\sum_{j=1}^{n+1} a_j-L|+|\sum_{k=1}^{n+2} a_k-L|<\dfrac{\epsilon}{3}+\dfrac{2\epsilon}{3}+\dfrac{\epsilon}{3}=\dfrac{4\epsilon}{3}##.

This proves ##\sum_{n=1}^{\infty} b_n## is convergent and that its limit is ##2L-a_1+a_2##

I haven't retried the other direction. Now I'll put the matter in my hands (brain). Thanks!
 
Last edited:
  • #4
mahler1 said:
It's true, the forward direction wasn't difficult:

Lets prove that ##\sum_{n=1}^{\infty} b_n=2L##

By hypothesis, ##\sum_{n=1}^{\infty} a_n## is convergent. Call its limit ##L##, then given ##\epsilon>0##, there exists ##N_{\epsilon} : n\geq N_{\epsilon} \implies |\sum_{i=1}^n a_i -L|<\dfrac{\epsilon}{3}##. Note that for ## n\geq N_{\epsilon}##, ##|\sum_{i=1}^n a_{i+1} -L|<\dfrac{\epsilon}{3}## and ##|\sum_{i=1}^n a_{i+2} -L|<\dfrac{\epsilon}{3}## (this could be proved by an index change of the form ##j=i+1## and ##j=i+2##).

Then, for ##n\geq N_{\epsilon}##, ##|\sum_{i=1}^n b_i -2L|=|\sum_{i=1}^n a_i -L + 2\sum_{i=1}^n a_{i+1} -2L+L-\sum_{i=1}^n a_{i+2}|\leq |\sum_{i=1}^n a_i -L|+2|\sum_{i=1}^n a_{i+1} -L|+|\sum_{i=1}^n a_{i+2}-L|<\dfrac{4\epsilon}{3}##. This proves ##\sum_{n=1}^{\infty} b_n## is convergent and that its limit is ##2L##.

I haven't retried the other direction. Now I'll put the matter in my hands (brain). Thanks!

Sort of, but you can't just reindex series like that without paying some attention to the terms you dropping by doing that. I think ##\sum_{n=1}^{\infty} b_n=2L-a_0+a_1##.
 
Last edited:
  • #5
Dick said:
Sort of, but you can't just reindex like that without paying some attention to the terms you dropping by doing that. I think ##\sum_{n=1}^{\infty} b_n=2L-a_0+a_1##.

You're right, but I think the limit is ##2L-a_1+a_2##, I suppose you were considering the first term of the sequence to start at ##n=0## and not ##1##. Now I correct my post. By the way, I still don't know how to prove the other implication, could you give me one more hint?
 
  • #6
mahler1 said:
You're right, but I think the limit is ##2L-a_1+a_2##, I suppose you were considering the first term of the sequence to start at ##n=0## and not ##1##. Now I correct my post. By the way, I still don't know how to prove the other implication, could you give me one more hint?

Yes, you are right. I was starting at n=0. Sorry. And I haven't even thought about going the other way yet. But I'm not going to do that until tomorrow. Yawn! That gives you some time to think about it and beat me to the punch.
 
  • #7
Dick said:
Yes, you are right. I was starting at n=0. Sorry. And I haven't even thought about going the other way yet. But I'm not going to do that until tomorrow. Yawn! That gives you some time to think about it and beat me to the punch.

Thanks for all the suggestions and corrections. I am going to bed now and see if I have some sort of brain illumination while sleeping.
 
  • #8
mahler1 said:
I still don't know how to prove the other implication, could you give me one more hint?
it's essentially the same. What is the difference between the two sums taken from 1 to r?
 
  • #9
haruspex said:
it's essentially the same. What is the difference between the two sums taken from 1 to r?

Your question led me to this:

Lets call ##s_n=\sum_{k=1}^n a_k##, ##t_n=\sum_{k=1}^n b_k##. I'll prove that ##\lim_{n \to \infty} 2s_n-t_n=a_1-a_2##. From this, one immediately could deduce that ##\{s_n\}_{n \in \mathbb N}## converges iff ##\{t_n\}_{n \in \mathbb N}## converges.

##2s_n-t_n=2\sum_{k=1}^n a_k-\sum_{k=1}^n b_k=2\sum_{k=1}^n a_k-\sum_{k=1}^n a_k-2\sum_{k=1}^n a_{k+1}+\sum_{k=1}^n a_{k+2}=2\sum_{k=1}^n a_k-\sum_{k=1}^n a_k-2\sum_{k=1}^n a_{k}-2a_{n+1}+2a_1+\sum_{k=1}^n a_{k}+a_{n+2}+a_{n+1}-a_1-a_2=2s_n-s_n-2s_n+s_n+(a_{n+2}-a_{n+1}+a_1-a_2)=a_{n+2}-a_{n+1}+a_1-a_2##.

Then, ##\lim_{n \to \infty} 2s_n-t_n=\lim_{n \to \infty} a_{n+2}-a_{n+1}+a_1-a_2=a_1-a_2##.

It follows that ##\{s_n\}_{n \in \mathbb N}## converges iff ##\{t_n\}_{n \in \mathbb N}## converges.
 
  • #10
mahler1 said:
Your question led me to this:

Lets call ##s_n=\sum_{k=1}^n a_k##, ##t_n=\sum_{k=1}^n b_k##. I'll prove that ##\lim_{n \to \infty} 2s_n-t_n=a_1-a_2##. From this, one immediately could deduce that ##\{s_n\}_{n \in \mathbb N}## converges iff ##\{t_n\}_{n \in \mathbb N}## converges.

##2s_n-t_n=2\sum_{k=1}^n a_k-\sum_{k=1}^n b_k=2\sum_{k=1}^n a_k-\sum_{k=1}^n a_k-2\sum_{k=1}^n a_{k+1}+\sum_{k=1}^n a_{k+2}=2\sum_{k=1}^n a_k-\sum_{k=1}^n a_k-2\sum_{k=1}^n a_{k}-2a_{n+1}+2a_1+\sum_{k=1}^n a_{k}+a_{n+2}+a_{n+1}-a_1-a_2=2s_n-s_n-2s_n+s_n+(a_{n+2}-a_{n+1}+a_1-a_2)=a_{n+2}-a_{n+1}+a_1-a_2##.

Then, ##\lim_{n \to \infty} 2s_n-t_n=\lim_{n \to \infty} a_{n+2}-a_{n+1}+a_1-a_2=a_1-a_2##.

It follows that ##\{s_n\}_{n \in \mathbb N}## converges iff ##\{t_n\}_{n \in \mathbb N}## converges.
Looks good.
 
  • Like
Likes 1 person

1. How do I prove the convergence of a series?

To prove the convergence of a series, you must show that the limit of the sequence of partial sums approaches a finite value as the number of terms increases.

2. What is the difference between absolute and conditional convergence?

Absolute convergence refers to when a series converges regardless of the order of its terms, while conditional convergence means that the series only converges when the terms are arranged in a specific order.

3. What are some common methods for proving convergence?

Some common methods for proving convergence include the ratio test, the root test, and the comparison test.

4. Can a series converge if its terms do not approach zero?

Yes, a series can still converge even if its terms do not approach zero. This is known as the divergence test, where the limit of the terms does not equal zero.

5. How does the comparison test work?

The comparison test compares a given series with another series whose convergence is already known. If the second series converges and the given series is smaller in magnitude, then the given series must also converge.

Similar threads

  • Calculus and Beyond Homework Help
Replies
2
Views
713
  • Calculus and Beyond Homework Help
Replies
2
Views
196
Replies
8
Views
1K
  • Calculus and Beyond Homework Help
Replies
6
Views
399
  • Calculus and Beyond Homework Help
Replies
1
Views
267
  • Calculus and Beyond Homework Help
Replies
3
Views
422
  • Calculus and Beyond Homework Help
Replies
5
Views
493
  • Calculus and Beyond Homework Help
Replies
5
Views
1K
  • Calculus and Beyond Homework Help
Replies
8
Views
2K
  • Calculus and Beyond Homework Help
Replies
34
Views
2K
Back
Top